Foundations of Reading Test

Pataasin ang iyong marka sa homework at exams ngayon gamit ang Quizwiz!

Read the passage below; then answer the question that follows. For the second time that week, Saul forgot to wash his hands after-working on his painting. He had gotten so involved filling in the ocean in his picture that he had barely even heard the teacher telling everyone it was time to put away their easels and wash up for lunch. He had put his supplies away, but, still thinking about the ocean, he had gone straight to his desk. Now he saw that he was leaving blue-paint handprints on his desk, on his shirt, on his books-even on his lunchbox. Estella looked over at him and joked, "Hey, Saul! You're the new Kind Midas! Only you turn everything to blue!" Saul rolled his eyes at her as he got back up to go to the sink. This passage would be most suited for helping students: A. recognize a literary allusion B. analyze story elements C. predict future events D. analyze an author's point of view

A. A literary allusion is when an author refers indirectly to an earlier literary work by using a name, word, or phrase closely associated with that literary work. In this passage, the character Estella refers to Kind Midas, a character of Greek legend who was given the power to turn everything he touched to gold. While a teacher could use this passage to analyze perhaps one story element, such as character (B), or possibly to predict future events (C), the passage is not particularly well suited to teach these skills. Also, in this passage the author renders the action objectively (describing a series of observable events in emotionally neutral language without commentary); thus, it would not lend itself to an analysis of the author's point of view (D).

Which of the following sentences contains a pair of italicized words that differ from one another by one phoneme? A. He took off his cap so that he could take a nap. B. She works at at bank that is located near the bank of a river. C. She told him not to buy a ticket because she had already bought one. D. His face looked pale after he carried the pail of water for a mile.

A. A phoneme is a phonological unit of language, a discrete speech sound in a particular language that native speakers of the language recognize as sufficient to distinguish between two phonologically similar but separate words. For example, the speech sounds /k/ and /n/ are phonemes in English because native English speakers who hear the pair of spoken forms /kap/and /nap/ regard them as distinct words. B is incorrect because the two identically spelled words bank (financial institution ) and bank (land form abutting a river) are pronounced the same way; i.e., they contain the identical sequence of phonemes. C is incorrect because the two words differ by more than one phoneme. D is incorrect because although the pare of words, pale and pail, differ in their spelling, they are identical in their pronunciation; i.e., they contain the same phonemes.

A teacher can most effectively support first graders' development of rapid automatic word recognition by first teaching students how to: A. apply consistent phonic generalizations in common words. B. use context clues to determine the meanings of words. C. identify the constituent parts of multisyllable words. D. look up unfamiliar words in the dictionary.

A. Automaticity is the rapid recognition of a word without conscious attention to the decoding process. Research indicates that accurate decoding skills are a prerequisite to the development of, and readiness to benefit from instruction in, automatic word recognition. Applying consistent phonics generalizations to decode common words is a foundational decoding skill appropriate for beginning readers at the first-grade level. B is incorrect because a reliance on context clues for word recognition. C is incorrect because dividing multisyllable words into constituent parts is an advanced skill typically taught after developing readers have achieved automaticity reading many single-syllable words. D is incorrect because looking up words in a dictionary is related to decoding only as a corrective strategy and does not present phonics patterns explicitly or systematically. It is therefore of limited effectiveness in developing accurate decoding skills and automaticity among beginning readers.

Which of the following criteria would be most important to consider when selecting "leveled texts" for use in assessments and guided reading with beginning-level readers? A. The texts should use repeated words and natural oral language structures. B. The texts should require readers to use problem-solving to connect text to illustrations. C. The texts should emphasize use of literary language and dialogues. D. The texts should feature a range of punctuation and context-specific vocabulary.

A. Beginning readers need lots of practice reading a variety of texts written at their independent reading levels to reinforce their knowledge of phonics and sight words already taught, build their automaticity, and improve their reading fluency. Leveled texts are series of texts constructed using controlled vocabulary and syntactic structures. Within a particular level, the texts share many of the same vocabulary words and follow basic syntactic structures, both of which gradually increase in difficulty in successive levels. Matching students to appropriate-level texts is key to providing beginning readers with effective practice opportunities. B is incorrect because the primary goal of using these texts with beginning readers is to reinforce students' decoding skills and promote reading fluency, not to promote their use of picture clues. They are also not designed to promote students' knowledge of literary language or devices (C). Because of their controlled nature, leveled texts feature a limited range of punctuation, and they purposefully introduce only a few new content vocabulary words per text (D).

A sixth-grade teacher is planning explicit instruction to help students read and understand sentences that have a complex sentence structure. Which of the following skills would be most effective for the teacher to focus on first? A. identifying the independent clause at the heart of a sentence B. distinguishing between sentences that use passive and active voice C. identifying common transition words that link ideas in two or more sentences D. distinguishing between demonstrative and indefinite pronouns in a sentence

A. Complex sentences contain two or more clauses, with the main clause being the independent clause that contains the sentence's subject and predicate. Identifying the main clause will help students begin to deconstruct a complex sentence to determine its missing. B is incorrect because a sentence written in passive voice does not necessarily contain multiple clauses. C and D are incorrect because identifying transition words or distinguishing between demonstrative and indefinite pronouns would not help students deconstruct a complex sentence.

A fifth-grade teacher is about to begin a new unit on weather and climate. Which of the following types of vocabulary words from the unit would be most appropriate for the teacher to pre-teach? A. words that are conceptually challenging B. high-frequency, phonetically irregular words C. multisyllable words D. high-frequency words with multiple meanings

A. Conceptually challenging words, especially those associated with a new content-specific unit of study, are not likely to be in students' oral vocabularies, let alone in their reading vocabularies. To support students' reading comprehension of content-area texts associated with the unit, it is critical to introduce students to key concepts and associated vocabulary in both their oral and written forms prior to reading. B and D are incorrect because high-frequency words, whether with irregular spelling or multiple meanings (B and D respectively), are likely to be in students' reading vocabularies. Similarly, students begin to encounter many multisyllable words (C) in their reading by second grade, so this feature alone would not necessarily indicate that such words would be unfamiliar to students.

A sixth-grade class is working on an Internet research project about various natural resources and their uses. The teacher could best support students' effective use of the Internet for their research by: A. providing students with a checklist of questions that prompt critical evaluation of information on Web sites. B. giving students a list of Web sites that have been preapproved based on the site's reading levels. C. encouraging students to search for Web sites that are easy to navigate and that contain familiar vocabulary. D. teaching students to employ a variety of search engines to locate relevant Web sites.

A. Determining whether a given Web site contains reliable information ia an important step in conducting research on the Internet. Having students question the validity of content they read on a Web site will build this skill, prompting their ability to conduct more effective and bias-free research. B is incorrect because giving students a list of preapproved Web sites does not promote their independent research skills. Encouraging them to rely only on user-friendly Web sites (C) could lead students to ignore Web sites that contain valuable, objective information. Employing a variety of search engines (D) does not necessarily lead to more effective research, since different search engines are likely to identify a similar list of potential Web sites.

Mrs. Lopez teaches second grade. After completing a science experiment, she gathers her students on the carpet to write a summary of what they learned. Mrs. Lopez asks students to help her record the responses on chart paper, sharing the marker. She provides guidance and sentence starters as needed. After everyone has recorded their responses, the class reads them aloud together. Which type of writing experience is Mrs. Lopez demonstrating? A. Interactive writing B. Shared writing C. Independent writing D. Guided writing

A. During interactive writing experiences, teachers and students work together to create writing pieces. Teachers and students share the writing utensils, with teachers guiding the students as they record their thoughts. In shared writing experiences, teachers record students' thoughts on paper. Students do not help with the writing. IN independent writing, students utilize the strategies they have learned to complete writing pieces independently. Guided writing occurs when teachers work with small groups of strategically grouped students on targeted writing skills.

Electronic reading books are advantageous for beginning or struggling readers primarily because this type of computer software: A. scaffolds learning by providing a high level of interactivity. B. helps students develop familiarity with reading from a computer screen. C. provides students with models of good reading practices and habits. D. minimizes the focus on written text by using sound effects and voices to convey meaning.

A. Electronic books can offer many scaffolds to beginning or struggling readers, including providing the opportunity to hear the pronunciation or the definition of an unfamiliar word in the text or to listen to the whole text read aloud by a proficient reader. While electronic books can be available for use on a computer, they are not necessarily limited to this medium (B). Electronic books can provide good models of fluent reading; however, they are inanimate objects, so they cannot model good reading practices or habits (C). While electronic books may use auditory input to enhance comprehension (D), this is intended to be a scaffold for the reader and not a substitute for the text.

Before beginning a new content-area reading passage, a fourth-grade teacher asks students to think of words related to the topic of the text. The teacher writes the words on the board and then asks the students to suggest ways to group the words based on meaningful connections. The teacher also encourages them to explain their reasons for grouping particular word together. This series of activities is likely to promote the students' reading development primarily by helping them: A. extend and reinforce their expressive and receptive vocabularies related to the text's topic. B. infer the meaning of new vocabulary in the text based on word derivations. C. strengthen and extend their understanding of the overall structure of the text D. verify word meanings in the text by incorporating syntactic and semantic clues into their word analysis.

A. Grouping words related to a text based on conceptual categories and the words' associative meanings helps deepen students' understanding of the vocabulary. Discussing and justifying connections among the words further enhances students' understanding of the words and promotes retention of new words. B is incorrect because, while exploring the etymology of target vocabulary words can also be an effective strategy for deepening students' understanding of the words, the series of activities described in the scenario did not involve exploration of word derivations. Nor did it involve using semantic or syntactic clues from the text to verify words' meanings (D). C is incorrect because studying the conceptual connections among key vocabulary in a text is not likely to enhance students' understanding of the text's structure.

Students in a third-grade class are working on an interdisciplinary unit on Native Americans of the Northeast. The teacher has selected a historical novel for students to read during the unit to help them gain insight into people's daily lives in a particular Native American nation at a particular point in time. However, the teacher is aware that the novel's text complexity may make comprehension difficult for a group of struggling readers in the class. Which of the following strategies would be most effective for the teacher to use to support the struggling readers' comprehension of the novel and their purpose for reading? A. engaging the students in guided reading and rereading of key passages in the novel B. having the students stop after reading each chapter and try to summarize key events of the plot in their own words C. asking the students to rewrite the story from the perspective of a different character in the novel D. encouraging the students to read key chapters of the novel aloud together by taking turns reading specific pages

A. Guiding students in reading and rereading key passages of a complex text scaffolds their understanding of the text by allowing them to delve deeper into the ideas presented in the text with each successive reading. The activities described in response B, C, and D would not be effective in facilitating struggling readers' comprehension of the novels because these activities do not provide the students with sufficient scaffolding to support students' construction of meaning of a complex text.

Which of the following best describes the primary advantage of having a student read a passage silently and then provide a "retelling" as a means of assessing the student's comprehension, rather than having the student answer questions? A. A retelling is open-ended and requires the student to construct a description of the passage more independently of the examiner. B. The results of a retelling are more objective and easier to quantify than the results of direct questioning. C. The procedure involved in retelling tends to be more familiar to a wider range of students, including English Language Learners. D. A retelling can provide information about the student's inferential comprehension skills, which questioning cannot provide.

A. Having a young student provide a "retelling" (or having an older student construct a summary) of a text removes clues or scaffolds from the assessment that teacher questions often provide and ensures that a student's responses are based solely on his or her reading of the text. Some disadvantages of retellings include that the results are not easy to quantify (B) and the procedure is often unfamiliar to students, particularly to students who are new to U.S. schools (C). Also, since retellings are unstructured and usually elicit only explicitly stated details about the text, this type of assessment is not necessarily effective in assessing a student's inferential comprehension skills (D).

Mr. Suarez teaches a preschool class containing students in the preliterate stage o writing. He wants to help his students understand the relationship between spoken and written words using an orchard. The group will record memories of their field trip in writing, and Mr. Suarez will read the writing back to students repeatedly throughout the week. Which activity would be most appropriate? A. Shared writing B. Independent writing C. Interactive writing D. Partner writing

A. In shared writing experiences, students share throughts during a class discussion, and the teacher records them on paper. Because students are in the preliterate stage of writing, they are not yet using sound/symbol relationships to spell words. Writing at this stage may resemble scribbling or contain strings of pretend letters, so Mr. Suarez can record the students' thoughts for the this activity so that the story can be reread multiple times. If students write independently or with partners, it will be difficult to maintain the message of the story each time it is reread. Similar difficulties would occur with interactive writing because students help write the shared story on paper.

A fifth-grade teacher guides students in reading a complex literary text. First, the teacher reads aloud the beginning of the text as the students follow along silently in their copies. Next, the teacher rereads key phrases and sentences, asking students what the author meant by certain statements or by the choice of certain words. Finally, the teacher and students reread the section aloud together with expression. The teacher repeats these steps with each section of the text. This activity promotes reading proficiency primarily by: A. modeling for students how to engage in close reading of academic texts. B. developing students' word consciousness and love of interesting new words. C. helping students achieve grade-level fluency benchmarks for accuracy and rate D. encouraging students to apply metacognitive comprehension strategies as they read

A. In the scenario, the teacher guides students' engagement in key processes involved in close reading, such as rereading a complex text to enhance comprehension, analyzing the author's use of language, and questioning the author's intentions and goals. B is incorrect because, while the activity does prompt students to think about the author's choice of words in certain sentences, the activity does not "primarily" focus on vocabulary. C is incorrect because the activity focuses more on building students'' comprehension, and their prosody by extension, rather than on developing their fluency with respect to accuracy and rate. D is incorrect because the strategies involved in the activity do not necessarily involve metacognition.

Which strategy can be used to build readers' confidence and increase both comprehension and fluency when introducing an unknown text? A. Introducing new vocabulary before reading B. Reading the first few pages aloud before asking students to finish it independently C. Listing what students want to learn about the topic D. Asking students to predict what the text will be about based on the title and cover

A. Introducing new vocabulary words will help students decode the words faster when they are encountered in the text, thus assisting with fluency. Recognizing the words and knowing their meanings will also build readers' confidence. Additionally, explaining the meanings of the words in advance will help students comprehend the text when they read it independently. reading the first few pages aloud without discussion will not help students recognize or understand the vocabulary words, nor will it highlight important words beyond the first few pages. Listing what students want to learn about the topic helps to set a purpose for reading, not explain vocabulary. Predicting helps students activate prior knowledge, make connections, and stay actively engaged in the reading process.

Which of the following strategies would be most effective in promoting kindergarten children's ability to recognize and name letters of the alphabet? A. The teacher says the name of a letter while the children each trace its shape on a cutout letter. B. The teacher posts the entire alphabet around the room in several different formats. C. The teacher reads aloud to the children from books that contain mostly words that follow regular phonics patterns. D. The teacher emphasizes the initial sounds of words when reading to the children.

A. Letter naming entails an ability both to distinguish between letters and to associate particular letter shapes with their names. The activity described in A simultaneously activates visual, auditory, kinesthetic, and tactile sensory pathways in learning letter shapes and associating these shapes with their names. Research suggests that such multisensory techniques are effective for this purpose. B, C, and D are incorrect because the activities described do not focus on one or both subskills required in letter naming. The activity described in B exposes students to letter shapes without promoting students' ability to distinguish the letters from one another or associate each letter shape with a letter name. The activities described in C and D do not present students with information about letter shapes or letter names.

Before reading aloud a book about a farm to a readers, a first-grade teacher has the students brainstorm and briefly discuss words related to farms. Next, the teacher reads the text aloud from a big book, pointing to each word being read. Periodically, the teacher stops to discuss with students key concepts or events described in the text and to guide students in relating the text to the illustrations. After finishing the read-aloud, the teacher puts the book in the classroom library and encourages the students to read it on their own. The most important reason for putting the book in the classroom library is to promote the students': A. love of reading by facilitating their access to a story that they have already heard, understood, and enjoyed. B. understanding of the alphabetic principle by reinforcing their knowledge of letter-sound correspondences. C. oral language development by providing them with the opportunity to imitate the teacher's reading of a text. D. use of metacognitive strategies by allowing them to practice self-monitoring when reading silently.

A. Reading aloud to children is a research-based strategy for promoting students' interest in reading and in books. Providing students with access to a book that the teacher has already read aloud and discussed with them is likely to foster their love of reading because the teacher has provided various scaffolds for comprehending the story, and thus they are likely to experience some level of success in reading it independently. Responses, B, C, and D are incorrect because simply providing access to the book is not likely to contribute to the students' knowledge of the alphabetic principle (B), oral language development (C), or use of metacognitive strategies (D), which all require explicit instruction and guided practice.

Over the course of the school year, a sixth-grade student who had been a fluent, proficient reader in previous years is having increasing difficulty comprehending grade-level literary and informational texts assigned in class. The results of informal, curriculum-based assessments indicate that the student still meets grade-level expectations in vocabulary knowledge, but the student's reading rate and comprehension have dropped below grade level. The student also tends to choose fiction and graphic novels written well below the sixth-grade level for independent reading. The student's overall reading performance suggests that the student would likely benefit most from instruction focused on promoting the student's: A. knowledge and skills related to understanding complex academic language. B. understanding of important features of skilled prosodic reading. C. development of decoding skills and automaticity recognizing grade-level sight words. D. skill in applying contextual analysis and other word analysis strategies.

A. Research has shown that independent reading of a wide range of literacy and informational texts play a key role in students' academic language development, as the language of books tends to be more sophisticated than that of everyday social language. The student in this scenario likely has adequate functional reading skills, given the students' prior reading proficiency. However, the student's independent reading is limited to "fiction and graphic novels written well below" grade level, thus limiting the student's exposure to grade-level academic language, which in turn inhibits progress in reading comprehension. B, C, and D are incorrect because the student had been fluent, proficient reader in previous years and continues to read low-level texts with pleasure, thus foundational reading skills are not likely at the root of the student's current difficulty with complex academic texts.

A second-grade teacher pairs students who are reading at approximately the same independent reading level for a partner-reading activity. During the activity, the two partners sit side by side and take turns reading aloud from a shared text. Over a period of several days, the partners read a large number of independent-level texts together. This activity is best designed to promote students': A. development of reading rate and automaticity. B. awareness of key aspects of prosodic reading. C. development of comprehension skills and strategies. D. awareness of new phonics elements

A. Research has shown that reading the same text several times builds comprehension and improves reading rate and automaticity with respect to the given text, but these gains do not necessarily transfer to other texts. The most effective way for students to improve reading rate and automaticity is to practice reading many texts written at their independent reading level (i.e., texts that they can read accurately). B is incorrect because, while improving one's reading rate may improve one aspect of prosody, it does not necessarily contribute to other aspects of prosody, such as appropriate phrasing and intonation. Clearly, the activity is not "best designed" to promote prosody. C is incorrect because the activity does not focus on developing students' comprehension skills and strategies. D is incorrect because the students are reading independent-level texts (i.e., texts that they can read with very high degrees of decoding accuracy and comprehension). Thus, it is unlikely that the texts contain "new" phonics elements (i.e., elements that have not yet been introduced to these students).

The words enjoyable, maneuverable, corruptible, and convertible best illustrate which of the following principles? A. The spelling of a suffix can vary depending on its root word. B. The accented syllable of a root word can shift when certain suffixes are added to it. C. The addition of a suffix can alter the spelling of its root word. D. The pronunciation of a suffix can change when added to certain root words.

A. The four words presented contain the same derivational suffix, which has two variant spellings -able and -ible. The correct spelling of the suffix in a particular word depends on the root of the word. B, C, and D are incorrect because the phonological and orthographic variations described in these responses do not occur in the words listed.

The teacher asks students to locate and mark places mentioned in the chapter on a map as they read. This activity is most likely to help students: A. use visualization to facilitate their comprehension of the text. B. paraphrase content to make the text more understandable. C. connect elements in the text to their background knowledge. D. identify the text's main ideas and supporting details.

A. The passage mentions several sites or landmarks with which students may be unfamiliar. Locating and marking these sites on a map would improve the students' ability to visualize the direction of the troop movements mentioned in the passage. B, C, and D are not skills that would be scaffolded, facilitated, or reinforced by having the students consult maps.

A fifth-grade student reads the sentence, "After playing with her friends all day, Kaylee did her science homework, her geography project, and her composition in all fell swoop." The student asks the teacher for help understanding what is meant by the phrase one fell swoop. The teacher can best help the student understand this idiomatic expression by: A. discussing with the student more examples of the phrase used in context. B. directing the student to look up different meanings of fell and swoop in the dictionary. C. helping the student create a tree diagram of the structure of the phrase. D. asking the student to find other sentences in the text that use the words fell and swoop.

A. The phrase one fell swoop is an idiom. An idiomatic expression is a sequence of words that has a specific meaning beyond the sum of the meanings of the component words. In addition to explaining the idiom's specific meaning, the most effective way to promote a student's understanding of a new idiom is to provide the student with several examples of the idiom used in comprehensible contexts. B and D are incorrect because one fell swoop functions semantically as a single unit, so using various means for determining the meanings of its component words would not help the student determine the meaning of the unit as a whole. Tree diagrams (C) are used to represent the grammatical structure of a sentence. This would be useful in helping the student determine the grammatical function of the phrase in the sentence but not its meaning.

A kindergarten teacher wants to promote students' understanding of the alphabetic principle. Which of the following would be the most effective first step in a sequence of instruction designed to achieve this goal? A. Talk with students about selected consonants using a series of posters that each feature one consonant and contain pictures of items whose initial phoneme demonstrates that consonant's sound. B. Have students trace both lowercase and uppercase letters of the alphabet and then practice reproducing the letters on their own. C. Talk with students about the title, beginning, middle, and end of a story and point to these parts while reading the story aloud from a big book. D. Put labels on several familiar objects in the classroom and regularly read the labels aloud to the students.

A. Understanding the alphabetic principle entails the recognition that letters and letter patterns used in writing an alphabetic language correspond to the sounds in the spoken words of the language. The activity described in A would promote understanding of the relationship between letters and the initial sound in familiar words. Focusing on the initial phoneme in these words reflects an understanding that segmenting an initial consonant is a relatively easy phonemic awareness skill that beginning readers have likely mastered and thus is appropriate to use din early instruction on the alphabetic principle. B is incorrect because the activity does not relate letters to the sounds in words. C is incorrect because it does not focus on the letters in a text or relate them to discrete sounds in words. D is incorrect because the individual sounds are not isolated in related to the letters in the printed word.

Use the information below to answer the three questions that follow. Before reading aloud a book about a farm to a group of beginning readers, a first-grade teacher has the students brainstorm and briefly discuss words related to farms. Next, the teacher reads the text aloud from a big book, pointing to each word being read. Periodically, the teacher stops to discuss with students key concepts or events described in the text and to guide students in relating the text to the illustrations. After finishing the read-aloud, the teacher puts the book in the classroom library and encourages the students to read it on their own. The students are most likely to be successful in their independent reading of the book if: A. the students can decode and understand the meaning of at least 95 percent of the words in the text. B. the text does not include compound sentences joined by coordinating conjunctions such as and, or, or but. C. the students come from homes where silent reading is extensively modeled and encouraged by caregivers. D. the text primarily deals with fictional rather than factual accounts of characters and/or events.

A. Word recognition and vocabulary knowledge are primary factors affecting reading comprehension. In addition to understanding a text's key vocabulary words orally, the reader must also be able to recognize those words in print. If the student could both decode and understand the meaning of at least 95 percent of the words in the text, they would be performing well above the threshold of 90 percent, at which point comprehension breaks down. The factors described in responses B, C, and D are not as significant as vocabulary knowledge in contributing to students' ability to read a text independently.

Which of the following statements best describes how oral vocabulary knowledge is related to the process of decoding written words? A. A reader applies decoding skills to unfamiliar written words in order to increase his or her oral vocabulary knowledge. B. A reader's oral vocabulary knowledge allows the reader to derive meaning as he or she decodes written words. C. A reader must have extensive oral vocabulary knowledge in order to learn decoding processes. D. A reader's oral vocabulary knowledge is dependent on his or her development of strong decoding skills.

B. A reader uses decoding skills to derive the correct pronunciation of a phonically regular word, but the phonological form alone does not prompt understanding of the word's meaning. To understand the word's meaning, the reader must already have the word in her or his oral vocabulary. A is incorrect because decoding an unfamiliar word correctly does not provide the semantic information about the word required to add the word to a reader's oral vocabulary. C is incorrect because decoding is the process of identifying the spoken form of a word based on its printed form, which can be done without knowledge of the word's meaning. D is incorrect because in the early stages of reading development, oral language precedes development of decoding skills.

A second-grade teacher reads a trade book aloud to the class. Which of the following postreading activities would be most likely to promote the students' comprehension of the story by enhancing their literary analysis skills? A. encouraging the students to identify the key vocabulary words in the story B. discussing with the students how the characters in the story respond to major events and challenges C. asking the students to reread the story silently and respond to several literal comprehension questions D. having the students "freewrite" about the story in their reading response journals

B. At the second-grade level, a significant focus of literary analysis involves thinking about characters and how characters' actions and words provide clues about their motivations. A, C, and D are incorrect because having students identify important vocabulary form the story (A), reread the story and respond to literal comprehension questions (C), or engage in a "freewrite" about the story (D) could enhance their comprehension of the story but would not develop their literary analysis skills.

During a small group activity, a teacher asks her students to repeat a CVC word slowly, stretching out the sounds. As students repeat the word, they slide one penny forward for each sound they hear. Which phonemic awareness skill are students practicing? A. Blending B. Segmenting C. Phoneme identification D. Phoneme isolation

B. By taking a whole word and breaking it into its individual sounds, the students are practicing segmenting. Blending is the opposite process, where students take individual sounds and combine them to form a whole word. Phoneme isolation refers to identifying the beginning, middle, or ending sound in a word. Phoneme identification refers to identifying a common sound in a group of words with either the same beginning, middle, or ending sound.

Students in a third-grade class are studying different forms of transportation that are used around the world. As part of this unit of study, they work together to create a semantic map of words associated with transportation, including words that they have recently learned (e.g., barge, rickshaw). This activity is most likely to promote students' vocabulary development by: A. showing them how structural analysis can be used to determine the meaning of new vocabulary. B. helping them to categorize, visualize, and remember new vocabulary. C. guiding them to discover the multiple meanings of new vocabulary. D. providing them with frequent, varied reading experiences using the new vocabulary.

B. In content-area subjects, concept building and vocabulary building are reciprocal and closely interdependent processes. Helping students construct a semantic map of words associated with a particular content-area topic such as transportation both deepens and broadens their understanding of the topic. It also provides them with a framework for making connections among related words (e.g., barge, rickshaw). Creating the visual display of the semantic map, in which related words are literally grouped together, also aids students' recall of the connections among the words. The other responses are incorrect because teaching students how to use structural analysis (A), encouraging exploration of multiple-meaning words (C), and providing students with multiple exposures to new words. (D) are all strategies for supporting vocabulary development but are not the primary goals of the semantic mapping activity described.

A second-grade teacher regularly reviews spelling patterns previously taught. The teacher also provides students with multiple opportunities to read and write connected text that features words containing the target spelling patterns and to engage in word sorts focused on previously taught spelling patterns. These types of activities are likely to promote students' reading proficiency primarily by developing their: A. knowledge of grade-level vocabulary B. reading fluency with respect to accuracy C. awareness of different types of morphemes D. word recognition with respect to sight words

B. In this scenario, the teacher uses both reading and writing activities to reinforce previously taught spelling patterns. Research has established that encoding and decoding are reciprocal processes and that spelling knowledge can contribute to word-reading accuracy. Accuracy is a key component of reading fluency. A is incorrect because the focus of the activity is on word recognition, not vocabulary acquisition. C is incorrect because the activities described do not relate to morphological awareness. Sight words are words that do not follow regular spelling patterns or that contain phonics elements that have not yet been taught, which makes D incorrect.

Two proficient readers are answering postreading comprehension questions about a chapter in a content-area textbook. *The first student demonstrates exceptional recall of details from the chapter but has difficulty answering questions about the gist of the chapter. *The second student can give an outstanding summary of the chapter but has difficulty remembering specific facts from the chapter. Which of the following best explains the most likely reason for the students' varied understanding of the text? A. The first student is more proficient than the second student at using metacognitive comprehension strategies to make sense of the text. B. Each student applied different reading comprehension skills when reading the text. C. The second student is more proficient at reading for literal understanding than for inferential understanding. D. Each student brought a unique set of prior experiences to the reading of the text.

B. Many factors contribute to reading comprehension. Skilled readers use different comprehension strategies to achieve different purposes. For example, the first student described in this item may have skimmed the chapter to look for specific words or phrases mentioned in the comprehension questions, while the second student may have scanned the chapter for main ideas but did not dwell on individual terms of facts. A is incorrect because the first student had difficulty answering questions about the gist of the chapter, so it is likely that that student did not apply metacognitive strategies to enhance understanding during reading. The second student was able to construct a strong summary of the chapter, which suggests an ability to use inferential skills, so C is incorrect. While background knowledge and experiences have a strong influence on reading comprehension (D), this factor cannot explain why the students focused on such different levels of the text.

A third-grade teacher periodically reads aloud from a chapter in content-area textbooks and describes his thought processes as he reads. Following is an example: "'The moon does not shine on its own. The sun's light reflects off the moon.' Hmm. I'm imagining that the sun is like a flashlight shining on the moon in the dark. 'As the moon rotates, only the part that faces the sun is visible form the Earth.' I'm not quite sure what "visible" means, but it sounds kind of like vision, which I know has to do with eyes. It probably means the part that we can see from the Earth. Now, that makes me wonder- why do we see different amounts of the moon at different times? Let's see if the next part of the chapter explains this..." This practice is most likely to promote students' reading proficiency by: A. exposing them to new vocabulary in context. B. modeling for them metacognitive comprehension strategies. C. giving them an example of fluent oral reading. D. summarizing for them the main ideas of an expository text.

B. Metacognitive reading comprehension strategies prompt students to think about their thinking as they read a text. The teacher models two powerful metacognitive strategies: visualizing to support comprehension and self-questioning to clarify understanding and to set a purpose for further reading. A is incorrect because while the teacher talks about the vocabulary word visible and determines what it means deductively, the focus of this portion of the teacher's think-aloud is promoting the students' comprehension of the sentence as a whole in which the word visible occurs. C is incorrect because the teacher pauses several times during reading to make comments about the text, so the teacher does not present an example of fluent oral reading. The teacher could conceivably summarize the text at the end of the think-aloud (D), but there is no evidence in the description to suggest this will occur.

Which of the following types of assessments would best provide information about the comparative reading proficiency of students in an elementary school? A. a test of vocabulary development B. a norm-referenced survey test C. a reading miscue inventory D. a diagnostic portfolio

B. Norm-referenced tests are designed specifically for the purpose of comparing students' performance. Norms are statistics that describe the test performance of a representative sample group. In general, the tests in A and C are designed to measure the degree to which students meet specific objectives (i.e., they are criterion-referenced); therefore, A and C are incorrect. A diagnostic portfolio (D) is individually administered and would most likely contain a comprehensive battery of reading assessments designed for diagnostic purposes.

A fifth-grade teacher gives students a reading guide for an informational text that they will be reading independently. The reading guide contains various activities, including prompting students to summarize certain passages, to explain relationships between concepts according to specific information in the text, and to determine the meaning of domain-specific words based on appositives or appositives phrases embedded in the text. This reading guide is likely to be most effective for achieving which of the following instructional purposes? A. developing students' ability to read their text evaluatively B. encouraging students to read and interact closely with the text C. supporting students' development of prosodic reading skills D. teaching students to adjust their reading rate based on text complexity

B. Reading guides such as the one described compel students to reread a complex academic text multiple times so they can complete the activities in the guide, which are specifically designed to help the students interact with the content and language of the text. A is incorrect because the activities do not focus on developing students' evaluative comprehension skills. C is incorrect because use of such a guide would not promote development of prosodic reading, since it is a silent reading activity that does not involve explicit or implicit teacher modeling of or instruction in prosodic reading. A text-based reading guide would not teach students how to adjust their reading rate based on text difficulty (D) because it focuses on just one specific text.

A English Language Learner pronounces tigers as tiger when reading the following sentence aloud. They saw tigers at the zoo. Which of the following actions is most appropriate for the teacher to take first in response to the student's miscue? A. guide the student to reading lists of nouns with and without plural -s on the end B. verify that the student understands that tigers means more than one tiger C. provide the student with independent practice in adding plural -s to singular nouns D. provide a picture cared to determine whether the student can identify a tiger

B. Since there is often a discrepancy between an English Language Learner's receptive and productive language skills, the student may in fact recognize the omitted element but simply not pronounce it, may not have the element in his or her oral vocabulary, or may have difficulty decoding the element. To assess the source of the error and respond appropriately, the teacher must first determine whether the missing element is in the student's oral vocabulary. A and C are incorrect because the teacher must ensure that the plural -s is in the student's oral vocabulary before beginning differentiated instruction in decoding words with the morpheme. D is incorrect because assessing the student's knowledge of the word tiger does not provide information relevant to the student's miscue.

Skimming is likely to be the most effective strategy for accomplishing which of the following reading tasks? A. evaluating the validity of information on on Internet Web site B. previewing a chapter in a content-area textbook C. synthesizing information from various sources for a research report D. studying specific facts for a content-area exam

B. Skimming involves a quick, superficial reading of a text to get an overall impression of the material. This would be an appropriate and effective strategy for previewing a textbook chapter. The other responses are incorrect because evaluating the validity of information on a Web site (A), synthesizing information from a variety of sources for research purposes (C), and studying facts for a content-area exam (D) all require a more careful reading of textual materials.

As a first-grade teacher reads a big book to a group of students, the teacher points to the beginning consonants of selected words and accentuates the sound the initial letter makes. This activity is most likely to promote the students': A. awareness of multisyllable words. B. ability to isolate individual sounds in words. C. structural analysis skills. D. ability to blend the sounds in words.

B. The activity described contributes to students' beginning reading development in a number of ways, including reinforcing their phonemic awareness (i.e., their ability to distinguish the separate phonemes in spoken word), their knowledge of letter-sound correspondences, and their understanding of the alphabetic principle. The other responses are incorrect because the activity focuses students' attention on only the first letter of the words, so the activity would not promote students' awareness of multisyllable words (A), their structural analysis skills (C), or their ability to blend all the sounds in a word (D).

Pointing out the title, beginning, middle, and end of a book to a group of preschool children before reading the book aloud to them contributes to their reading development primarily by promoting their: A. understanding of text directionality. B. development of book-handling skills. C. understanding of the concept of schema. D. development of literal comprehension strategies.

B. The instruction the teacher provides before reading aloud, in conjunction with how the teacher holds and proceeds through the book during reading, helps beginning readers learn how to hold a book with the front cover facing up and the spine on the left, and then how to move through the pages from front to back. A is incorrect because the teacher does not show students lines of text or indicate to them how to track print during reading. C is incorrect because the teacher does not attempt to access students' background knowledge, or schema, related to the text. D is incorrect because the activity does not focus on teaching students strategies for comprehending the text.

A teacher can best help sixth graders draw inferences from informational text by asking them to complete which of the following statements? A. In my opinion... B. The passage suggests... C. In comparision... D. The author's first point is...

B. To draw an inference from a text is to derive a conclusion that is not stated explicitly in the text but rather is suggested by facts or premises presented. A is incorrect because stating an opinion about a text does not necessarily involve deciding or deriving a conclusion from explicit information in the text. C and D focus on the use of literal comprehension rather than inferential comprehension.

Which of the following is the most important reason for a fourth-grade teacher to assign a variety of high-quality trade books as a component of reading instruction? A. The themes typical of children's literature tend to reinforce students' development of literal comprehension skills. B. Reading across genres helps students develop an understanding of the structures and features of texts. C. Simplified syntax and controlled vocabulary provide necessary scaffolding for student who are struggling readers. D. Reading diverse texts helps to promote students' development of phonological and phonemic awareness skills.

B. Trade books are books marketed to a general audience, as opposed to books developed for specialized audiences. Children's trade books cover a range of topics and include both fiction and nonfiction texts. Wide reading of trade books provides students with exposure to many genres and helps them begin to develop an awareness of the structural differences between genres. Trade books are not limited to specific themes (A), nor do they use simplified syntax or controlled vocabulary (C). Reading trade books does not explicitly (or implicitly) teach fourth-grade students phonological and phonemic awareness skills (D), which are typically developed at a much earlier grade level using primarily oral language activities.

An advantage of using assessment tools such as portfolios and scoring rubrics is that they: A. provide more objective results than do multiple-choice tests. B. promote student participation in self-assessment activities. C. ensure consistency among different evaluators. D. offer more reliable assessment data.

B. Typically, the development of student work portfolios involves students in selecting and self-assessing some or all of the materials that are included in their individual portfolios. Students can use scoring rubrics to self-assess and guide their work. The other responses are incorrect because students and teachers can develop their own criteria for selecting student work for a particular portfolio and can develop their own scoring rubrics for evaluating that work; thus, the results of this form of assessment are neither more objective (A), nor more consistent (C), nor more reliable (D) than the results of standardized assessments.

Considerations of validity in test construction relate most closely to : A. how a particular examinee's test performance relates to a pre-established standard. B. whether the test questions effectively measure their specified content. C. how a particular examinee's test performance of other examinees. D. whether the test results are likely to be repeatable with a similar examinee test group.

B. Validity is the degree to which a test measures what it is intended to measure. A is incorrect because criterion referencing is the term used when a student's performance is related to a pre-established standard or set of objectives. C describes norm referencing. D describes test reliability.

The following sentence is missing several words. (1) unusual (2) of spices (3) the soup an (4) flavor. A word with the suffix -tion would fit best in which of the blanks in the sentence? A. (1) B. (2) C. (3) D. (4)

B. Words ending with the derivational suffix -tion are nouns (e.g., combination, addition). The syntax of English determines the word orders that are possible (i.e., grammatically correct) in sentences. For example, articles and adjectives precede the nouns they modify. In the sentence, unusual is an adjective, so blank 1 would most likely represent an article (e.g., the, an) and blank 2 would most likely represent a noun. Therefore, blank 2 is an appropriate placement for a word ending in -tion. A is incorrect because blank 1 most likely represents an article. C is incorrect because it is the only likely placement for the verb in this sentence, given the structure of the sentence frame. D is incorrect because blank 4 follows an article and precedes a noun, so it is not a likely position for another noun.

Read the sentence below; then answer the question that follows. My family went to the circus last weekend. I liked the clowns the best. They were very funny. A student makes several miscues when reading these sentences aloud. Which of the following miscues represents an error in decoding consonant blends? A. omitting circus B. pronouncing clowns as clones C. saying bet for best D. shortening funny to fun

C. A consonant blend is a sequence of two or more consonants in a word, each of which represents a separate phoneme. For example, the sequence of consonants at the end of the word best represents the sequence of phonemes /s/ and /t/. A student who says bet for best is omitting the letter s, an error in decoding the consonant blend at the end of the word. The other responses are incorrect because these miscues are unrelated to decoding consonant blends. A is a whole-word omission. B represents an error decoding a diphthong. D is an omission of an entire syllable.

Which of the following students demonstrates variation in reading development that would require intervention focused on explicit phonics instruction? A. A kindergarten student who can recite the alphabet from memory but has difficulty distinguishing individual phonemes in words. B. A first-grade student who can easily decode nonsense words but has limited comprehension of the meaning of text. C. A second-grade student who is adept at using context clues to identify words but has difficulty sounding out the letters in unfamiliar words. D. A third-grade student who can read most grade-level text fluently but has difficulty with unfamiliar irregular low-frequency words.

C. A second-grade student who lacks the fundamental skill of sounding out letters has phonics skills well below grade level and is at significant risk for reading failure. Thus, an appropriate intervention for this student would be explicit phonics instruction. A is incorrect because this student has not yet developed phonemic awareness, so an intervention in explicit phonics would be too advanced. B is incorrect because the student's ability to decode nonsense words demonstrates development of phonics skills. D is incorrect because a student who has attained fluency reads accurately and with comprehension, which is overwhelming evidence that the student has accurate decoding skills-including strong phonics skills.

A fifth-grade class is about to read a play about the life of Harriet Tubman called "Travels on the Railroad." Which of the following prereading activities would best promote students' comprehension of the text? A. introducing the common elements of plays as a genre and looking at sections of a printed play as a class B. asking students to generate several questions about the play based on the play's title C. asking students to share what they already know about the time period D. encouraging groups of students to create and perform their own short skits about the same subject

C. Activating students' prior knowledge related to a text is a powerful strategy for promoting their comprehension of text. A is incorrect because it develops students' general knowledge of the genre, but it does not facilitate the students' comprehension of the text, which is based on historical events with which they may be unfamiliar. Similarly, asking questions about a text based on its title (B) can be very helpful in setting a purpose for reading and thus supporting reading comprehension; however, the title of this particular play is metaphorical, so it may generate questions that are totally irrelevant. Creating original skits based on the same subject (D) also can extend students' comprehension of a text, but this strategy would be most effective as a postreading activity rather than as a prereading activity.

Sixth-grade students have just finished reading a chapter in a novel and are getting ready to write an entry in their response journals. The teacher could most effectively develop students' literary response skills by assigning which of the following journal prompts? A. What new vocabulary words did you learn when reading this chapter? List and define the new words from the chapter. B. What happened in the chapter? Describe two or three events from the chapter. C. What do you think is the main ideas or theme of the novel? Relate specific events in this chapter to the theme you suggest. D. Which characters are mentioned in this chapter? List each of the characters.

C. At the secondary level, formal literary response involves developing a thesis adn providing evidence from the text to support the thesis. To help sixth graders learn how to construct an effective literary response to a text, the teacher should provide opportunities, such as prompted writing in response journals, to develop a thesis related to a text (e.g., identifying a main idea or theme of the text), and then to support their thesis by citing evidence from the text. A emphasizes vocabulary development, while the questions in responses B and D focus students on literal comprehension skills only-that is, identifying factual information explicitly stated in the text.

A preschool teacher is reading a story to his class. As he reads, he holds the book so the children can see the words and pictures while his finger follows the line of print. This activity would contribute to the children's reading development primarily by: A. promoting their development of letter recognition skills. B. helping them recognize phonemes that occur frequently in print. C. developing their awareness of left-to-right directionality. D. promoting their understanding of letter-sound correspondence.

C. By following the line of print with his finger while reading, the teacher provides students with a visual demonstration that print is read from left to right. A, B, and D are incorrect because the teacher does not explicitly identify or indicate component letters, phonemes, or letter-sound correspondences related to the text.

Which of the following strategies would be most effective in promoting second graders' decoding of multisyllable words? A. giving students opportunities to read literature that offers repeated exposure to predictable text B. prompting students to sound out the individual phonemes that compose multisyllable words C. encouraging students to compare the parts of new multisyllable words with known single-syllable words D. reinforcing students' recognition of high-frequency multisyllable words using drills and flashcards

C. By the second grade, students have typically learned a wide variety of syllable patterns in single-syllable words. Since most of the syllables in multisyllable words follow the same patterns as those in single-syllable words, the primary challenge for students just learning to decode multisyllable words is learning to recognize the words as a series of discrete syllables. This recognition allows students to apply their prior knowledge of syllable patterns to decoding longer words. The strategy described in C is effective because it focuses students' attention on recognizing the component syllables in these words. A is incorrect because reading predictable text is typically used with beginning readers and is not appropriate for teaching decoding of multisyllable words. B is incorrect because sounding out words letter by letter is a strategy beginning readers can use to decode very simple words with one-to-one letter-sound correspondences, but it is not an efficient strategy for decoding the complex multiletter phonics patterns typically encounter by second-grade readers. D is incorrect because practicing with flashcards-focusing on rapid recognition of words-would not be appropriate until students have learned to process all of the letters in target words systematically and can decode the words with a high degree of accuracy.

Which of the following strategies would be most appropriate to use to promote second-grade students' ability to analyze key ideas and detail in a literary text? A. explicitly teaching students the key features and conventions of different literary genres B. prompting students to evaluate the significance of a story's setting with respect to its theme C. helping students create a story map of the main characters in a story and the events with which they are involved D. encouraging students to clarify their understanding of a story by reflecting on their personal experiences

C. Characters and plot are key elements of story structure in narrative texts. At the second-grade level, students' literary analysis focuses primarily on these elements. Helping students create a story map focused on main characters and events would both reinforce their understanding of these key components of story grammar and support their analysis of key ideas and details of the text. A is incorrect because learning about the features of different literary genres helps support reading comprehension but it would not necessarily promote students' ability to analyze key ideas and details in a particular text. B is incorrect because, while setting can be important to a narrative text, characters and plot are generally more significant, especially with primary-grades texts. D is incorrect because it focuses more on students' personal responses to a text rather than on their development of literary analysis skills.

A first-grade teacher designs the following activity. 1. Divide students into pairs. 2. Have students sit back-to-back. 3. Give one student in each pair a picture of a familiar object to describe to his or her partner. 4. The partner tries to name the object based on the description. This activity is likely to contribute to students' literacy development primarily by: A. helping them begin to make connection between print and the spoken word. B. fostering their ability to work independently of teacher guidance. C. promoting their oral language development and listening comprehension. D. encouraging them to practice speaking skills.

C. IN the activity described, the students must convey the identifying features of an object using oral language alone. This prompts the students to use descriptive language and synonyms to try to evoke a mental image of the object in the minds of their partners. Conversely, the partners must listen attentively and focus carefully on these oral language clues to visualize and make accurate deductions about the objects. A is incorrect because the activity in completely oral, so it would not be effective in promoting students' connections between spoken language and print. Since the activity is structured and monitored by the teacher, it is also not likely to enhance the students' ability to work independently (B). D is incorrect because a critical component of the activity is to promote students' listening comprehension.

According to basic principles of research-based, systematic phonics instruction, which of the following common English letter combinations would be most appropriate for a first-grader teacher to introduce first? A. ir B. kn C. th D. oi

C. In research-bsed, systematic phonics instruction, phionics elements are introduced according to their utlity for beginning readers, and therefore according to their frequency of use in words appearing in primary-grade texts. Among the letter combinations given, th occurs most frequently in such texts and therefore is among the very first letter combination taught. A, B. and D are incorrect because these letter combinations appear significantly less frequently in primary-grades texts than th does.

Instruction in structural analysis is likely to promote upper elementary students' reading comprehension primarily by: A. facilitating their ability to use phonics generalizations to decode words. B. enhancing their familiarity with the text structures and features used in different genres. C. equipping them with strategies for understanding the meaning of unfamiliar multisyllable words. D. increasing their knowledge of key vocabulary found in content-area textbooks.

C. In the context of reading, structural analysis is the process of recognizing the morphemic structure of words. Readers who recognize component morphemes (i.e., base words, prefixes, suffixes, and inflections) in unfamiliar, morphologically complex words can use this knowledge to support decoding or to derive the likely meaning of the words. A is incorrect because instruction in word analysis skills as a decoding strategy is typically taught during earlier stages of reading development. B is incorrect because recognizing component morphemes in words does not provide information relevant to text structures and features. D is incorrect because teaching the use of structural analysis is not sufficient in itself to help students learn key content-are vocabulary.

Which of the following informal assessment results provides the clearest indication that a kindergarten child has attained a beginning level of phonemic awareness? A. The student can clap the "beats" or syllables of familiar multisyllable words. B. The student can delete the second "word" or syllable in compound words. C. The student can identify the beginning sound of single-syllable words. D. The student can substitute phonemes in the medial position of single-syllable words.

C. Phonemic awareness is a specific type of phonological awareness involving the ability to distinguish the separate phonemes in a spoken word. Identifying the beginning sound of a single-syllable words is typically one of the earliest phonemic awareness skills developed, while substituting medial phonemes (D) is considered a more advanced phonemic awareness skill. A and B describe skills at the syllable or word level and are therefore not considered phonemic awareness skills.

Which of the following types of activities would be most important to include on a daily basis when planning reading instruction for first graders who are developing as beginning readers? A. activities that introduce students to basic concepts about print B. activities that emphasize listening to and producing rhyming, alliteration, and similar forms of wordplay C. activities that promote students' development of decoding and other word analysis skills D. activities that emphasize memorization of lists of grade-level-appropriate sight words.

C. Phonics and other word analysis skills, such as learning common inflectional endings and the orthographic rules governing their addition to words, are critical skills that promote beginning readers' development of proficiency in decoding. These skills in turn support students' development of reading fluency and comprehension. Daily, systematic, differentiated instruction and practice in these essential skills are necessary to ensure all students in a first-grade class develop into proficient readers. While some students at this level may still require differentiated or remedial instruction in concepts about print (A) and /or basic phonological awareness skills (B), these skills are generally mastered by most students in kindergarten, so they would not be taught to a whole first-grade class on a daily basis. Instruction in sight words is an important component of a research-based beginning reading program that is primarily phonics based; however, memorization of lists of sight words on a daily basis (D) is an ineffective instructional approach.

Before asking students to read a new fiction book independently, the teacher conducts a picture walk with the class. What is the teacher's primary goal for conducting the picture walk? A. Encouraging the use of syntactic cues B. Setting a purpose for reading C. Activating students' prior knowledge D. Developing students' metacognitive skills.

C. Picture walks are frequently used to activate students' prior knowledge. When readers look at the pictures, they make connections between the content of the new texts and what they already know from other texts or life experiences. These connections can spark interest in the stories and deepen comprehension. Teachers may encourage the use of syntactic cues during guided reading groups, but this is not typically part of a picture walk. Setting a purpose for reading is another beneficial prereading activity, but it is not always a part of a picture walk either. Metacognitive skills, which assist students with self-monitoring during reading, are also beneficial, but they are not typically the focus of a picture walk.

Alex is a first-grade student. His teacher notices that he commonly substitutes words that make sense in the sentences but don't match the print. For example, he recently said, "The care drove down a street, " while the text actually said, "The car drove down the street." Which cueing system is Alex using? A. Graphophonic B. Semantic C. Syntactic D. Pragmatic

C. Readers use the syntactic cueing system when they consider sentence structure and grammar to decode unknown words. Alex is substituting a word that sounds right in the sentence. If Alex were using the graphophonic cueing system, he would choose a word that is structurally similar to the existing word, such as a word that starts with the same sound. If Alex were using the semantic cueing system, he would use picture cues or prior knowledge to guess the word. The pragmatic cueing system refers to consideration of the purposes of reading in given situations.

In order to select a trade book that emphasizes predictability, a teacher should ensure that: A. the text includes some pictures or illustrations. B. the concepts in the text are at an appropriate level of difficulty for the target student(s). C. a phrase, rhyme, or sentence is repeated throughout the text. D. the length of the text is not likely to exceed the attention span of the target student(s).

C. Reading aloud predictable texts to pre-alphabetic and emergent readers is an effective way to promote young children's motivation to read and their development of concepts of print. The portion of the text that is predictable is generally a phrase, rhyme, or sentence that is repeated throughout the text, much like the refrain of a song. This repeated text is easy for young children to learn quickly, which allows them to "read" along with the teacher. A, B, and D all describe other important features common to books written for young children. However, these are not features that make the books predictable.

If a student test is said to lack reliability, the test: A. is not measuring what it is supposed to measure. B. has not proven to be useful as an instructional intervention C. gives fluctuating scores in different administrations. D. has poor predictive value relative to students' classroom performance.

C. Reliability indicates the degree to which a test yields consistent results over successive administrations. If a test yielded fluctuating results, it would be considered to have low reliability. A relates to a test's content validity, while D relates to a test's predictive validity. Standardized tests are not intended to be used as interventions (D).

Which of the following strategies is likely to be most effective in promoting reluctant readers' interest in independent reading outside of school? A. Calculate numerical scores based on the number and difficulty level of the books students read at home and integrate the score into students' report card grade for reading. B. Encourage parents to give their children simple external rewards for at-home reading, such as an extra helping of a favorite treat. C. Encourage students and parents to read books together on a regular basis, either silently or aloud, and discuss their personal responses to each chapter or key event. D. Recommend that parents make their children' s daily television-watching time contingent on their reading a specified number of pages first.

C. Reluctant readers are often reluctant to read independently at home because they have experienced some level of difficulty in reading. Encouraging students to read with a proficient reader, such as a parent or guardian, and to discuss key events in the text helps scaffold and support their comprehension of the text. Sharing personal responses to books with a partner also underscores that reading can be a highly social and enjoyable act; this in turn promotes positive dispositions toward reading. A and B are incorrect because providing extrinsic rewards to students for their reading may work as a short-term strategy with some students, but it is not effective in promoting a lifelong interest in or love of reading. Withholding a treat or reward until reading is complete (D) is another short-term strategy, but it may actually increase reluctant readers' negative feelings about reading instead of fostering their positive feelings.

A fifth-grade class silently reads an informational text. in subsequent informal assessments, several students are able to read the text orally with fluency but they demonstrate poor overall comprehension of the text. The teacher could most appropriately address these students' needs by adjusting future instruction in which of the following ways? A. using informational texts that are written at the students' independent reading level B. providing the students with explicit instruction in grade-level-appropriate test-taking strategies C. introducing a text's key vocabulary and guiding the students in close reading of key passages D. emphasizing reading skill-building activities that focus primarily on narrative texts

C. Since the students could read the text orally with fluency during subsequent informal assessments, the teacher needs to consider other factors that may have affected the students' comprehension of the text. Informational texts introduce vocabulary words that are likely to be unfamiliar to many students in a fifth-grade classroom. Informational texts may also include academic language structures with which students are unfamiliar. For students to be able to read and comprehend such a text independently, the teacher may need to model and provide students with practice in close reading of key passages of the text, as well as explicitly teach key vocabulary prior to reading. Using only informational texts that are written at students' independent reading levels (A) is not an effective option, as these texts are unlikely to include important grade-level vocabulary and thus may not match grade-level standards for that content area. Similarly, simply providing students with instruction in test-taking skills (B) or focusing on narrative texts for reading activities (D) would not help students develop the content-specific knowledge and vocabulary required at their grade level.

A sixth-grade student encounters the following sentence in a short story. She experienced a sense deja vu as she walked down the street of the strange new city. The student asks the teacher about the meaning of deja vu in the sentence. The teacher could best respond by advising the student to take which of the following steps? A. Make note of the word in a vocabulary log, and then study the word after finishing the story. B. Use context clues in the sentence to guess the meaning of the word, and then try out that meaning in the sentence. C. Look up the word in the dictionary, and then paraphrase the sentence using the dictionary definition. D. Break the word into its component parts, and then compare the parts to the meanings of similar known words.

C. Stopping to consult a dictionary is generally disruptive to the reading process; however, in some situation, as with the sentence shown in the box, consulting a dictionary is the only effective method for determining the meaning of an unfamiliar term is a good method for both reinforcing understanding of the new term and returning the reader to the flow of the story. The other strategies given would not be effective in this particular situation. B is incorrect because contextual analysis would not be a useful strategy here, since the sentence around the term deja vu provides little or no clues as to the term's meaning. Using structural analysis (D) would also not be helpful, since the term is made up of two words and they are borrowed from the French language. Finally, simply noting the term in a vocabulary log and coming back to it after finishing reading the story (A) would cause the student to miss the significance of the sentence and could diminish the student's comprehension of the story.

A text includes the word indefensible, which is unfamiliar to some students in a fourth-grade class. Which of the following strategies for teaching the word would be most effective in both clarifying the meaning of the word and extending the students' vocabulary development? A. Have the students enter the word in their ongoing list of new vocabulary words then look up its definition independently. B. Use explicit explanation to describe the meaning of the word to the students before they read the text. C. Help the students apply structural and contextual analyses to construct and confirm the word's meaning. D. Ask the students to paraphrase the sentence that contains the word by substituting a synonym for the word.

C. Teaching students to use structural analysis and their knowledge of familiar English morphemes (i.e., the root defense and the affixes in- and -ible) to deduce the meaning of a new word containing these morphemes provides students with a powerful independent word-learning strategy. This strategy immediately extends students' understanding of both the target word and other words that contain these morphemes. Afterwards, the students should use contextual clues in the text to verify that the meaning they have deduced fits the context. A, B, and D are incorrect because they describe strategies students can use to clarify the meaning of a particular word, but they do not extend the students' vocabulary development beyond the target word.

Before reading aloud a book about a farm to a group of beginning readers, a first-grade teacher has the students brainstorm and briefly discuss words related to farms. Next, the teacher reads the text aloud from a big book, pointing to each word being read. Periodically, the teacher stops to discuss with students key concepts or events described in the text and to guide students in relating the text to the illustrations. After finishing the read-aloud, the teacher puts the book in the classroom library and encourages the students to read it on their own. The theoretical basis for including the brainstorming activity in this lesson is that having the students share their vocabulary knowledge about farms prior to the reading will: A. give the teacher an opportunity to assess and compare the students' oral language skills. B. reinforce the students' understanding and recognition of key concepts about print. C. facilitate the students' comprehension by activating prior knowledge and building schema. D. prepare the students to benefit from explicit phonics instruction related to the text.

C. The brainstorming activity described in the stimulus paragraph (on page A-57) is designed to activate students' prior knowledge and build background knowledge (schema) to facilitate their reading comprehension of the story. A is incorrect because the activity would not be very effective in assessing and comparing students' oral language skills, since their vocabularies are likely to vary widely on this topic (farms) depending on their prior experiences with farms. The activity also does not relate to the development of concepts of print (B) or phonics (D).

An English Language Learner reads academic texts fluently in her primary language but is struggling to understand her content-area textbooks in English. This student would likely benefit most from engaging in which of the following activities? A. translating textbook reading assignments from English into her primary language. B. receiving reading comprehension instruction with texts written in her primary language. C. learning to use metacognitive reading strategies with English text D. reading texts in her primary language that cover the same material as her English textbooks

C. The student reads academic texts well in her primary language, so she likely has both adequate background knowledge in the content areas and basic reading skills. The student would benefit from using metacognitive strategies such as self-monitoring to connect what she has already learned in the primary language with the content presented in the English textbooks. A, B, and D all focus on primary-language materials. While these strategies may be helpful in supporting the students' ongoing conceptual development with regard to specific content, they do not promote the students' ability to comprehend content-area textbooks in English.

A third-grade teacher has been conducting a series of ongoing assessments of a student's oral reading. Shown below is a sentence from a text, followed by a transcription of a typical example of the student's oral reading performance. Text: Her boots crunched through the snow. Student: Her boats crucked throw the snow. After reading the sentence, the student paused and then reread it without the teacher's prompting and self-corrected the errors. Based on this information, the teacher could best meet this student's needs by adjusting instruction in order to: A. enhance the student's oral vocabulary development. B. develop the student's ability to self-monitor comprehension. C. improve the student's decoding skills. D. promote the student's abliltiy to track print.

C. The student's oral reading performance in this sample strongly suggests a lack of foundational knowledge in phonics and sight words. The miscues indicate serious decoding difficulties with various phonics elements, including lack of automaticity in decoding common vowel digraphs (reading boats for boots), common consonant digraphs (reading ck for ch), and complex consonant clusters (reading crucked for crunched). The student also misread a high-frequency sight word (through) that should have been mastered by the end of second grade. Conversely, this oral reading sample provides no evidence that the student has difficulty tracking print (D). And, by self-correcting his or her errors without prompting, the student demonstrates both understanding of the text's vocabulary (A) and the ability to self-monitor for comprehension (B).

A fifth-grade teacher plans to have students read a chapter about the American Revolutionary War form their social studies textbook. The following is an excerpt from the chapter. The Battle of Bunker Hill took place on June 17, 1775. At the time, the American army occupied the area from Cambridge to the Mystic River. American troops gathered in Cambridge Common on the evening of June 16, 1775, and set out for Bunker Hill. Upon reaching Bunker Hill, however, officers decided to move to Breed's Hill, a smaller hill closer to Boston. Based on the excerpt from the chapter, which of the following graphic organizers would best promote students' awareness of the chapter's text structure? A. outline B. Venn diagram C. timeline D. sematic map

C. The text structure of this passage is chronological, focusing on when troops movements took place leading up to the battle. Timelines are the most effective form of graphic organizer for conveying a chronology of events. Outlines (A) and semantic maps (D) are well suited to passages that are organized into different sections or categories of related content. Venn diagrams (B) visually display similarities and differences between two or more related topics.

Which of the following principles is best illustrated by the words watched, wanted, and warned? A. Spelling is often the best predictor of the pronunciation of a suffix. B. Open syllables are usually pronounced with at long vowel sound. C. The spelling of a suffix is often more reliable than its pronunciation. D. The second letter of a consonant blend is usually pronounced as the onset of the following syllable.

C. The words listed all contain the regular past-tense inflection -ed. The ending is spelled the same way in all three cases, but the ending is pronounced differently in each word. In the word watched, the -ed ending is pronounced [t]. In the word wanted, the -ed ending is pronounced [ed]. IN the word warned, the -ed ending is pronounced [d]. A is incorrect because the three words illustrate that a single spelling of the ending can be pronounced three different ways. B is incorrect because one of the words contain an open syllable and thus do not illustrate characteristics of open syllables. D is incorrect because the syllabication guideline given is relevant to only one of the words (the word wanted).

A third-grade class that includes several English Language Learners is about to read a text about water sports. Which of the following teaching strategies would be most effective in promoting the English Language Learners' comprehension of the text? A. Have the students look up unknown English words using bilingual dictionaries and then make vocabulary lists in both languages. B. Pair English Language Learners with native speakers of English and have the native speakers explain any unknown vocabulary. C. Activate students' prior knowledge about the topic and provide visual aids such as illustrations and clarify new vocabulary. D. Give students a list of new vocabulary with definitions and ask the students to try to construct their own sentences using the words.

C. Two of the most effective instructional strategies for promoting English Language Learners' comprehension of a content-area text are activating the students' prior knowledge of the text's content and contextualizing new concepts and vocabulary related to water sports, the use of visuals such as illustrations and photographs would be the most effective (and efficient) strategy to use and would also activate any prior knowledge the students had regarding the topic. The strategies described in A, B, and D are not likely to be particularly effective in promoting English Language Learners' comprehension of the text because they neither build nor activate background knowledge related to the topic, and they do not help contextualize new vocabulary for learners.

A second-grade reading teacher notices that her students are decoding words accurately but struggle with appropriate phrasing and expression. Which activity would most likely help her students improve in this area? A. Introducing new texts of varied genres for students to read aloud independently B. Listening to audio versions of texts C. Leading students in repeated choral readings of familiar texts D. Participating in partner-reading experiences

C. When students reread familiar texts, they do not need to exert energy on decoding unfamiliar words. As a result, more energy is available to focus on fluency, including phrasing and expression. Choral reading allows students to match their reading rate and expression with others in the group who are modeling fluent reading, including the teacher. Reading new texts requires students to spend more energy on decoding and comprehension, which may interfere with fluency. Audio versions of texts can serve as models of fluent reading, but students still need opportunities to practice. Partner reading may be helpful if students are matched with fluent readers, but this is not guaranteed.

Which activity would be most appropriate to teach decoding a CVCe words? A. Covering parts of the word B. Chunking C. Building word families D. Blending

C. Word families are an effective strategy for teaching CVCe words. Once students have determined the sound made by the last three letters using knowledge of the silent e rule, they can identify other rhyming words within the same family by changing the first letter. This increases the number of words they can easily decode. Because these words have a silent letter that affects the vowel sound, covering parts of the word, chunking, and blending are not as effective.

A second-grader teacher has students pull two single-syllable nouns from a hat (e.g., bulb, light) and asks them to form words by putting the words together (e.g., lightbulb). Students then draw pictures to illustrate their new words and write short stories using the new words. This activity is likely to be most effective for helping students: A. use visualization as a reading comprehension strategy. B. apply knowledge of phonics generalizations. C. use context clues to identify unfamiliar words D. understand the concept of compound words.

D. A compound word is a word composed of two base words, and the meaning of a compound word is related to the meaning of these component parts. Combining two words to form a single new word illustrates the concept of compound words. A is incorrect because the activity described is not directly related to reading and comprehending a text. B is incorrect because the activity described does not focus on phonics patterns. C is incorrect because the activity is not relevant to reading or identifying unfamiliar words in connected text.

A sixth-grade teacher gives students several persuasive essays that present contrasting opinions on a current social issue. The teacher than asks students to consider the following questions as they read the texts. 1. What is the author's opinion on the issue? 2. How might the author's background influence his or her opinions? 3. What evidence does the author use to support his or her opinion? These questions are likely to be most effective for helping students: A. monitor comprehension of informational texts. B. identify the theme in expository texts. C. draw inferences from informational texts. D. analyze point of view of expository texts.

D. A persuasive essay is an expository text in which an author takes a stand on an issue, and tries to persuade readers by presenting an argument and evidence to support that stand. The teacher's questions are designed to help students analyze the authors' point of view by identifying an author's opinion as presented in an argument, evaluating how effectively the author supports the argument, and considering factors that have have influenced the author's opinion. A is incorrect because, while students may monitor their comprehension on the texts, comprehension monitoring is not explicitly taught or reinforced by the questions. Similarly, the skills described in B and C may be secondary outcomes of this activity but are not the primary goal.

Michael is a sixth-grade student who struggles to summarize fictional texts he has read. His summaries are often lengthy and include many unimportant details. Which type of graphic organizer could best help Michael develop concise summaries? A. KLW charts B. Semantic maps C. Venn diagrams D. Story maps

D. A story map helps students identify the main evetnts of a fictional story, along with the conflict and resolution. Listing the main events can assist Michael with writing a concise summary. A KWL chart helps students identify what they know, want to know, and learned about a topic. A semantic map contains a word of concept in the center of a diagram, with related words or phrases branching off from the middle. It is useful for helping students identify relationships between concepts. A Venn diagram helps students compare and contrast two or more things.

Which of the following text features are students likely to find most useful when previewing informational texts such as library books for a research project? A. index B. bibliography C. glossary D. table of contents

D. A table of contents shows how the content of a text is organized and provides clues to the types of information the reader is likely to find in each section. Skimming a text's table of contents provides a reader with a quick and effective preview of the text's content. This allows a reader to determine whether the text might be useful for a particular research project. A is incorrect because a typical index lists all the topics covered by a text in great detail, but it is organized alphabetically rather than grouped in broad categories, so it is less effective as a tool for previewing texts. B and C are incorrect because bibliographies provide information about the sources the author(s) used in developing the text, while glossaries provide definitions of key terms used in the text.

After students in sixth-grade class finish reading a historical novel about the U.S. Civil War, the teacher asks each student to bring in an object, that, to them, represents the book. The students must also identify a passage or passages from the book that they can use to support their choices when they present their objects to the class. This activity is most likely to promote students' reading development by helping them understand the importance of: A. determining an author's stated or implied main point of view. B. using text structure to develop a general summary of a literary work. C. identifying a novel's mood by analyzing the author's use of figurative language. D. basing interpretations about a literary work on textural evidence.

D. By asking students to think about an object that represents the novel to them and to identify passages from the text to support their choice, the teacher prompts the students to reflect on ideas and/or feelings the book evoked in them as they read and to ensure that their reflections are in fact derived from events in the text and not from their own experience or imagination. A is incorrect because the assignment is open-ended-individual students' interpretations of the book and their choice of an object are unique, so the activity does not require the students to focus on the author's point of view. It also does not necessarily focus their attention on the plot's sequence of events (B). C is incorrect because the activity does not explicitly require students to explore the author's use of figurative language.

Which of the following children is most in need of immediate intervention? A. a preschool child who has limited book-handling skills B. a kindergarten child who has limited ability to correlate alphabet letters with the sounds they make C. a first-grade student who still reads texts composed of single-syllable regular words and common sight words D. a second-grade student who still decodes words letter by letter

D. By second grade, students should be reading many regular words and sight words automatically. In addition, when reading new words that follow regular phonics patterns already taught, the students should be reading these patterns as chunks. A second-grade student who still decodes each word letter by letter is substantially behind grade-level benchmarks for oral reading fluency and requires immediate intervention. A preschool child who has limited book-handling skills (A), a kindergarten child who has not developed letter-sound correspondences for all letters (B), and a first-grade student who reads mostly decodable texts (C) are all performing within the average acceptable range for the respective grade levels.

A second-grade student demonstrates automaticity decoding grade-level regular and irregular words. However, the student frequently experiences poor text comprehension. Which of the following is the first step the teacher should take in order to promote this student's reading proficiency? A. evaluating the student's ability to apply grade-level phonics skills. B. determining the rate of the student's phonological processing C. evaluating the degree to which the student uses syntactic clues D. determining the extent of the student's vocabulary knowledge

D. For a student to comprehend a text during reading, the student must be able to decode each word and then connect the decoded word to a word in his or her oral vocabulary. Research has shown that if a reader does not understand the meaning of at least 90 percent of the words in a text, comprehension breaks down. Therefore, a student who has automaticity decoding grade-level words but a very limited oral vocabulary may be able to apply accurate decoding skills when reading an extended text but still not be able to make sense of the text. A and B are incorrect because the student in the scenario already demonstrates automaticity decoding grade-level regular and irregular words, so phonologically based deficits are not likely causing his or her comprehension difficulties. C is incorrect because readers use syntactic clues as a strategy for determining a word's grammatical function in a sentence. This can be helpful as a decoding strategy for verifying the meaning and pronunciation of a multiple-meaning word but is not a text-comprehension strategy.

What is one benefit of implicit phonics instruction? A. It leads to stronger decoding skills. B. Its progression from part to whole increases proficiency more quickly. C. Research suggests it is the most effective approach to phonics instruction. D. Phonics skills are taught in a meaningful context.

D. Implicit phonics instruction uses a whole-to-part approach, with students reading whole texts rather than starting with isolated phonemes. Students learn to recognize whole words by sight. Through analyzing and comparing words, they then discover phonics and spelling patterns. A benefit is that students read authentic text and learn in a meaningful context rather than practicing skills in isolation. However, research has shown that explicit phonics instruction, which progresses from part to whole, leads to stronger decoding, spelling, and comprehension skills.

Which of the following sets of words would be most effective to use when introducing students to the concept of structural analysis? A. late, great, wait, eight B. afraid, obtain, explain, remain C. swim, swims, swam, swum D. pretest, retest, tested, testing

D. In the context of reading, structural analysis is the process of recognizing the morphemic structure of words. Typically, structural analysis is introduced to students in the early elementary grades as a strategy for identifying words with inflections or simple affixes that are in students' oral vocabulary. The list in D features a phonically regular base word and affixes that are appropriate for beginning readers. A is incorrect because the words in this list contain only one morpheme and thus are not useful for demonstrating structural analysis. B is incorrect because the list features words that contain prefixes and roots that are inappropriate for beginning instruction in structural analysis. C is incorrect because this list features a base word (swim) with irregular past tense forms (swam, swum) which do not lend themselves to structural analysis.

A second-grade teacher administers spelling inventories periodically to help assess students' phonics knowledge. The following shows one student's performance on a spelling inventory at the beginning of the school year and again several months later. Dictated Word Student spelling Dictated Word Student Spelling set set set set star ster star star drive driv drive drive peach pech peach peche turn tarn turn turn join joyn join joyn The student's performance on the second administration of the spelling inventory indicates that the student made the most improvement in which of the following areas? A. initial and final consonants B. short vowels and diphthongs C. digraphs and blends D. long and r-controlled vowels

D. Patterns of improvement across the assessments suggest the student has learned two conventional spelling patterns. First, by correctly spelling star and turn in the second assessment, the student demonstrates progress in spelling words with r-controlled vowels. Second, by learning to use the VCe long-vowel pattern to spell the words drive and peach, the student demonstrates knowledge of a conventional spelling pattern for long vowels. Although the student's spelling of peach in the second assessment is incorrect, it nevertheless demonstrates progress in learning conventional spelling patterns used in English. A, B, and C are incorrect because the student shows no change over time in the indicated spelling skills.

A second-grade teacher notices that one of her students lacks fluency when reading aloud. The first thing the teacher should do in order to help this student is assess whether the student also has difficulties with: A. predicting B. inferring C. metacognition D. decoding

D. Reading fluency is integral to reading comprehension. The key indicators of reading fluency are accuracy, rate, and prosody. In the primary grades, the most common factor disrupting fluency is weak decoding skills, which most directly affects reading accuracy but affects the other indicators as well (e.g., by causing a slow rate or resulting in choppy reading). A, B, and C are incorrect because predicting, inferring, and metacognition are skills or strategies that support reading comprehension. Having difficulty in these skills or strategies would not necessarily affect a student's reading fluency.

Which of the following best describes the relationship between word decoding and reading comprehension in a beginning readers' development? A. Decoding skills and reading comprehension skills tend to develop independently of one another. B. Reading comprehension skills directly facilitate the development of decoding skills. C. Development of decoding skills is secondary to the development of reading fluency and comprehension skills. D. Rapid automatic decoding skills help facilitate development of reading fluency and comprehension

D. Reading research has shown that accurate decoding skills are a prerequisite to effective development of word-reading automaticity, which, in turn, is foundational to the development of reading fluency-that is, reading text accurately, at a rate that supports comprehension, and with speech-like phrasing and expressiveness. Furthermore, convergent research suggests that a lack of automatic decoding skills is a frequent cause of comprehension difficulties among students in the primary grades. A is incorrect because research indicates that poor decoding skills hamper comprehension and the development of reading comprehension skills. B and C are incorrect because being able to decode a text is foundational to understanding the text's meaning.

A third-grade teacher observes that students who read aloud fluently also demonstrate greater comprehension of expository texts. The best explanation for this is that fluent readers: A. possess a self-awareness that allows them to use metacognitive skills efficiently. B. have already developed the base of background knowledge typically covered by textbooks. C. have well-developed skills for decoding any level of text word by word. D. are able to focus their full attention and cognitive resources on the meaning of a text.

D. Research has shown that fluent readers have higher levels of comprehension than readers who lack fluency. By the third grade, fluent readers have developed automaticity in decoding, which allows them to focus on the meaning of what they are reading rather than on expending all their effort and energy on decoding each individual word letter by letter. Achieving reading fluency does not necessarily mean that students know how to use metacognitive skills (A) or that they have already developed background knowledge at the level of a textbook (B). Having fluency also does not mean that a third-grade student is skilled enough to decode any level of text (C).

Which of the following provides the best rationale for incorporating spelling instruction into a first-grade reading program? A. Spelling promotes phonemic awareness by teaching students to break words into onsets and rimes. B. Spelling facilitates vocabulary development by introducing students to new words. C. Spelling simplifies the reading process by focusing students on a limited set of decoding rules. D. Spelling supports word recognition by helping students learn and retain common phonics patterns.

D. Research has shown that the development of spelling and phonics skills is reciprocal; when instruction in these two skills is aligned, development of each skill area is reinforced and enhanced. Thus, learning the spelling patterns of words reinforces learning of their phonics patterns and supports word recognition. A is incorrect because phonemic awareness and the phonological awareness skill of onset-rime segmentation precedes and contributes to students' spelling development, not the reverser as stated in A. B is incorrect because most words targeted in spelling instruction are not new to students but already part of their oral vocabulary. C is incorrect because spelling knowledge continually expands the range of students' decoding skills and does not limit or simplify the decoding process.

A student who has mastered which of the following skills along the phonological awareness continuum is best prepared to begin explicit phonics instruction? A. being aware that a word is made up of one or more phonemes B. being able to separate a word's onset and rime C. being aware that words can be divided into syllables D. being able to segment and blend a word's phonemes

D. Research indicates that phonemic awareness skills, particularly phonemic blending (i.e., combining a sequence of speech sounds to form a word), are prerequisite skills to effective phonics instruction. A is incorrect because the skill described indicates phonemic awareness in the early stages, and not the fully developed skills that are prerequisite for instruction in phonics skills. B and C are incorrect because the phonological awareness skills described develop before the development of phonemic awareness, which is a prerequisite to phonics instruction.

One of the most important purposes of a standardized Informal Reading Inventory (IRI) is: A. to establish how prior knowledge and text organization influence a student's reading comprehension. B. to determine how a student uses semantic, syntactic, and other text clues to deduce a word's meaning. C. to analyze how a student's silent reading comprehension is influenced by oral reading fluency. D. to establish a student's independent, instructional, and frustration reading levels.

D. Standardized Informal Reading Inventions (IRIs) are administered individually to students to establish the students' reading levels. The results of this type of assessment provide evidence to guide the selection of reading materials for students for instruction and/or interventions and to guide students in their selection of materials for independent reading. A is incorrect because the results of IRIs do not provide information about how a text's organization or a student's prior knowledge affect the student's comprehension. IRI results are also not helpful in determining what types of clues students use to determine the meaning of unfamiliar vocabulary (B) or in analyzing the relationship between a student's silent-and oral-reading proficiencies (C).

Having kindergarten children practice tracing the letters of the alphabet in sand is most appropriate for children who are having difficulty: A. internalizing the alphabetic principle. B. recognizing the print carries meaning. C. understanding the relationship between spoken and written language. D. developing letter formation skills.

D. The activity described has children use arm movements and highly textured material to heighten their awareness of letter shapes and the sequence of strokes for forming letters. The activity is effective for students who are having difficulty with letter formation because it simultaneously activates visual, kinesthetic, and tactile sensory pathways in learning letter shapes and associating these shapes with their names. Research suggest that such multisensory techniques are effective for this purpose. A is incorrect because this activity does not address the relationship between the alphabetic writing system of English and the sounds of spoken language B is incorrect because the function of print is not addressed in this activity. C is incorrect because oral language is not addressed in this activity.

In which of the following sentences is context most helpful in understanding the italicized word? A. Tulip trees are ubiquitous in Virginia and in some other parts of the United States as well. B. John's friends surreptitiously planned a housewarming party for him soon after he had moved in. C. Mary is magnanimous in all of her dealings with people, even when she does not know a person well. D. Peter's mother was adamant that he should attend college, but his father did not seem to care.

D. This sentence provides a reader with both syntactic and semantic clues as to the meaning of the italicized word adamant. First, the coordinating conjunction but sets up a comparison between the two independent clauses, which immediately alerts the reader that the meaning of the second clause contracts the meaning of the first clause. Next, the meaning of the second clause is simple and clear-Peter's father did not seem to care whether or not Peter attended college. Thus, we now know that Peter's mother must have felt very strongly that he should attend. Indeed, someone who is adamant about something is unshakable or unmovable in his or her belief. In the other responses, the contexts do not provide adequate clues for the reader to deduce the meaning of the italicized words. In fact, a reader could substitute a variety of words for the italicized word in A (e.g., rare, blooming), B (e.g., immediately, collaboratively), or C (e.g.,

A fourth-grade English Language Learner is new to a school. Assessments suggest that the student can read orally with accuracy on grade level; however, the student's comprehension of grade-level textbooks fluctuates widely. Which of the following steps would be most appropriate for the teacher to take first in order to determine the cause of the student's difficulty? A. Assess the student's word analysis and decoding skills. B. Determine whether the student has a specific learning disability that affects language processing. C. Assess the student's level of first-language literacy. D. Determine whether the student has adequate vocabulary and background knowledge to support comprehension of the textbooks.

D. Vocabulary and background knowledge are critical components of reading comprehension, providing the foundation of the schema a student uses to construct meaning from the text. An English Language Learner's English vocabulary is likely to have gaps, particularly with respect to content-area topics that are not typical subjects of everyday conversations. An English Language Learner may be familiar with a content-area topic, but he or she may lack the relevant English vocabulary needed to comprehend and English-language text about the topic. In the scenario described, word analysis and decoding skills are not likely the cause of the student's difficulty (A), since the English Language Learner is able to read with grade-level accuracy. For the same reason, a learning disability that affects language processing (B) and the student's level of first-language literacy (C) are not likely factors impeding his or her reading comprehension.

A preschool teacher shows a group of children pictures of everyday objects. Below each picture is printed the letter of the alphabet that corresponds to the word's initial sound. As the teacher points to each picture, she names the object, then she points to the letter underneath it and says the sound it makes. The teacher invites the children to repeat the sound with her. This activity is likely to contribute to the children's reading development primarily by: A. illustrating the concept of word boundaries. B. focusing on auditory discrimination skills. C. introducing the concept of onset and rime. D. demonstrating that phonemes are represented by letters.

D. in the series of steps described (i.e., pronouncing a word, pointing to the letter that represents the initial sound of the word, and saying the sound the letter makes), the teacher demonstrates that familiar words have component sounds that can be pronounced in isolation, and that letters of the alphabet represent the component sounds (phonemes) of spoken words in print. A is incorrect because the teacher points out neither spoken nor written word boundaries in this lesson. B is incorrect because auditory discrimination entails comparing and distinguishing between two different sounds, whereas only a single sound is presented at a time in this lesson. C is incorrect because the teacher points out only the initial sound of each word and does not point out or discuss the remaining portion (the rime) of the word.

Define the graphophonic cueing system

Graphophonic cues involve the letter-sound or sound-symbol relationships of language. Readers identifying unknown words by relating speech sounds to letters or letter patterns are using graphophonic cues. ... They need to be encouraged to think about what word would make sense and fit in the sentence pattern or context.

What is consistent phonics generalizations?

Phonics Generalizations are phonics rules that are taught to emerging readers to help them learn letter combination sounds to increase reading and spelling ability. Phonics generalizations are NOT spelling rules. Phonics generalizations focus on patterns that words share. During a word sort, spellers group word features by certain categories.

Define semantic cueing system

Semantic cues refer to the meaning in language that assists in comprehending texts, including words, speech, signs, symbols, and other meaning-bearing forms. Semantic cues involve the learners' prior knowledge of language, text, and visual media, and their prior life experiences.

Define syntactic cueing system

Syntactic cues involve word order, rules and patterns of language (grammar), and punctuation. For example, the position a word holds in a sentence will cue the listener or reader as to whether the word is a noun or a verb.

Define the pragmatic cueing system

The pragmatic system provides information about the purposes and needs the reader has while reading; it governs what the reader considers important and needs to understand.

While reading a book about animals, a student struggles to decode the word giraffe. He then points to the picture and says, "Those are giraffes. I saw them at the zoo." Which cueing system is the student using to figure out the unknown word? A. Semantic B. Syntactic C. Graphophonic D. Pragmatic

The student is using both the picture in the book and his prior knowledge to make sense of the text, which demonstrates use of the sematic cueing system. When using the syntactic cueing system, readers select words that sound right using knowledge or grammar and sentence structure. When readers use the graphophonic cueing system, they use knowledge of letter/sound relationships to decode words. When using pragmatic cues, readers consider their purposes for reading in given situations.

Define orthography

orthography \or-THAH-gruh-fee\ noun. 1 a : the art of writing words with the proper letters according to standard usage. b : the representation of the sounds of a language by written or printed symbols. 2 : a part of language study that deals with letters and spelling. An orthography is a set of conventions for writing a language. It includes norms of spelling, hyphenation, capitalization, word breaks, emphasis, and punctuation.


Kaugnay na mga set ng pag-aaral

Chapter 4: Policy riders provision etc.

View Set

Chapter 3 - Real Estate Financing Principles

View Set

Professional Practice End of Chapter Questions

View Set

Chapter 26 & 27 Urinary System and Water Balance

View Set

Perception: Subliminal Perception

View Set

IT 301 Chp 2, CIS133 - Chapter 1 Quiz

View Set